Circles tell the area

Geometry Level 5

Circles O 1 , O 2 , O 3 , O 4 O_1, O_2 ,O_3, O_4 intersect at points A 1 , B 1 , A 2 , B 2 , A 3 , B 3 , A 4 , B 4 A_1, B_1, A_2, B_2, A_3, B_3, A_4, B_4 , as shown above. Points A 1 , A 2 , A 3 , A 4 A_1,A_2,A_3,A_4 lie on a circle pictured above. Also, we are given that the circumradius of triangle B 1 B 2 B 3 B_1 B_2 B_3 is 5, and that the product of the lengths of the sides B 1 B 4 , B 3 B 4 , B 1 B 3 B_1 B_4, B_3 B_4 , B_1 B_3 are all 40.

Find the area of triangle B 1 B 3 B 4 B_1 B_3 B_4 .


The answer is 2.00.

This section requires Javascript.
You are seeing this because something didn't load right. We suggest you, (a) try refreshing the page, (b) enabling javascript if it is disabled on your browser and, finally, (c) loading the non-javascript version of this page . We're sorry about the hassle.

1 solution

Manuel Kahayon
Jan 10, 2017

The key to this problem is knowing that B 1 , B 2 , B 3 , B 4 B_1, B_2, B_3, B_4 is cyclic, which can be easily proved using inversion. Denote A k A'_k to be the point which A k A_k maps to after inversion.

Inverting through A 1 A_1 maps the two circles which include A 1 A_1 into two sides of a triangle with vertices B 1 , A 2 , A 4 B'_1, A'_2, A'_4 . Since this inversion must map the circumcircle of A 1 A 2 A 3 A 4 A_1A_2A_3A_4 into a line, therefore, A 3 A'_3 must lie on line A 2 A 4 A'_2A'_4 .

The two circles which include A 3 A_3 must then map into two circles which pass through ( A 2 , A 3 ) , ( A 3 , A 4 ) (A'_2, A'_3),(A'_3, A'_4) , respectively. The circles intersect lines A 1 A 2 A_1A'_2 , A 1 A 4 A_1A'_4 at B 2 B'_2 and B 4 B'_4 respectively, and intersect at B 3 B'_3 . By simple angle chasing, we can see that B 1 B 2 B 3 B 4 B'_1B'_2B'_3B'_4 is a cyclic quadrilateral, therefore reinverting gives us the desired result.

This implies that the circumradius of the triangle B 1 B 3 B 4 B_1B_3B_4 is also 5.

And, the formula for the area of a triangle is given by

A r e a = A B C 4 R Area = \frac{ABC}{4R} , where A B C ABC denotes the product of the sides, and R R denotes the circumradius.

Our answer is then 40 ( 4 ) ( 5 ) = 2 \frac{40}{(4)(5)} = \boxed{2} .

I think "easily proved using inversion" is nontrivial enough you need to expand your thinking.

Jason Dyer Staff - 4 years, 5 months ago

Log in to reply

There we go :D

Manuel Kahayon - 4 years, 4 months ago

Log in to reply

Even with "simple angle chasing" you need to show what's going on. Note that our standard here is not the same as writing a proof in a paper; someone is not knowing how to solve the problem, so they read the solution. I know the cyclic quad problem has been given alone where people are stumped how to do the angle chasing. It's not "hard" but you should still show the steps.

Jason Dyer Staff - 4 years, 4 months ago

0 pending reports

×

Problem Loading...

Note Loading...

Set Loading...